Just make it least

Algebra Level 4

If a a , b b , c > 0 c > 0 , a 2 = b c a^2 = bc and a + b + c = a b c a + b + c = abc , then find the least value of a 4 + a 2 7 a^4 +a^2 -7 .


The answer is 5.

This section requires Javascript.
You are seeing this because something didn't load right. We suggest you, (a) try refreshing the page, (b) enabling javascript if it is disabled on your browser and, finally, (c) loading the non-javascript version of this page . We're sorry about the hassle.

5 solutions

Chew-Seong Cheong
Sep 16, 2016

Since a 2 = b c \color{#3D99F6}{a^2 = bc} a + b + c = a b c = a a 2 = a 3 \implies a+b+c = a\color{#3D99F6}{bc} = a\cdot \color{#3D99F6}{a^2} = a^3 . Then a b + b c + c a ab+\color{#3D99F6}{bc}+ca = a b + a a + c a = ab+\color{#3D99F6}{aa}+ca = a ( b + a + c ) = a a 3 = a 4 =a(b+a+c) = a\cdot a^3 = a^4 . Therefore, a 4 + a 2 = a b + b c + c a + b c \color{#3D99F6}{a^4} + \color{#D61F06}{a^2} = \color{#3D99F6}{ab+bc+ca} + \color{#D61F06}{bc} . Since a , b , c > 0 a, b, c > 0 , we can use AM-GM inequality as follows:

a b + b c + c a + b c 4 a 2 b 3 c 3 4 a 4 + a 2 4 a 8 4 = 4 a 2 a 4 3 a 2 a 2 3 a 4 9 \begin{aligned} ab+bc+ca+bc & \ge 4\sqrt[4]{a^2b^3c^3} \\ a^4 + a^2 & \ge 4\sqrt[4]{a^8} = 4a^2 \\ \implies a^4 & \ge 3a^2 \\ a^2 & \ge 3 \\ a^4 & \ge 9 \end{aligned}

a 4 + a 2 7 9 + 3 7 = 5 \begin{aligned} \implies a^4 + a^2 - 7 & \ge 9 + 3 -7 = \boxed{5} \end{aligned}

Well written , upvoted sir.

Priyanshu Mishra - 4 years, 8 months ago
Priyanshu Mishra
Sep 17, 2016

By AM-GM, we have

a + b + c 3 a b c 3 a + b + c \ge 3\sqrt [ 3 ]{ abc } .

Squaring and cancellation alongwith the fact a + b + c = a b c a + b + c = abc , gives

( a b c ) 2 27 { \left( abc \right) }^{ 2 }\ge 27 .

Putting a 2 = b c a^2 = bc gives

b c 3 bc\ge 3 or a 2 3 a^2 \ge 3 .

So, a 4 + a 2 7 ( 3 ) 2 + ( 3 ) 1 7 = 5 a^4 + a^2 -7 \ge { \left( 3 \right) }^{ 2 } + { \left( 3 \right) }^{ 1 } - 7 = \boxed{5} .

Nicely done. Are you appearing for GMO?

Kushagra Sahni - 4 years, 9 months ago

Log in to reply

No, i have given KVS JUNIOR MATHEMATICAL OLYMPIAD. If i get selected , then i can directly give INMO 2017.

Priyanshu Mishra - 4 years, 8 months ago

Log in to reply

Can you post the paper?

Kushagra Sahni - 4 years, 8 months ago

Log in to reply

@Kushagra Sahni Yes. Within half an hour.

Priyanshu Mishra - 4 years, 8 months ago
Mark Hennings
Sep 17, 2016

We have b , c b,c must be the roots of the quadratic X 2 ( a 3 a ) X + a 2 = 0 X^2 - (a^3 - a)X + a^2 \; = \; 0 For this quadratic to have two (possibly repeated) positive roots we need a 3 a > 0 a^3 - a > 0 and also ( a 3 a ) 2 4 a 2 0 (a^3 - a)^2 - 4a^2 \ge 0 , so that we must have a > 1 a > 1 and 0 a 6 2 a 4 3 a 2 = a 2 ( a 2 + 1 ) ( a 2 3 ) 0 \le a^6 - 2a^4 - 3a^2 = a^2(a^2+1)(a^2-3) and hence we must have a 2 3 a^2 \ge 3 . This makes the answer 9 + 3 7 = 5 9+3-7=\boxed{5} .

Utkarsh Dwivedi
Oct 11, 2016

Given that a 2 = b c , a + b + c = a b c a^2=bc,a+b+c=abc Second equation implies, b + c = a ( b c 1 ) b+c=a(bc-1) Applying AM-GM and substituting value of b c bc from first equation at the top,we have, a ( a 2 1 ) 2 > = a a 2 > = 3 \frac{a(a^2-1)}{2}>=a\Rightarrow a^2 >= 3 Putting the minimum value we got of a 2 a^2 we get answer as 5 \boxed{5} .

Josh Banister
Sep 19, 2016

Since we have a 2 = b c a^2 = bc , we can write b b and c c parametrically as b = k a b = ka and c = a k c = \frac{a}{k} for some variable real positive constant k k (as a , b , c > 0 a,b,c > 0 . Substituting this into a + b + c = a b c a+b+c = abc , we get a + k a + a k = a ( k a ) ( a k ) = a 3 a + ka + \frac{a}{k} = a(ka)(\frac{a}{k}) = a^3 . Since a > 0 a > 0 , we must have 1 + k + 1 k = a 2 1 + k + \frac{1}{k} = a^2 . Therefore for any k k , we can generate a value for a a and thus values for b b and c c .

To minimise a 4 + a 2 7 a^4 + a^2 - 7 , this is a strictly increasing function in a a for a > 0 a > 0 so to find the minimum of this, we need the minimum value for a a (or more over, a 2 a^2 ). Since we can make a 2 a^2 as small as possible by making 1 + k + 1 k 1 + k + \frac{1}{k} as small possible too, using the AM-GM inequality, we have 1 + k + 1 k 3 1 × k × 1 k = 3 1 + k + \frac{1}{k} \geq 3\sqrt{1\times k \times \frac{1}{k}} = 3 where equality holds when k = 1 k = 1 . This means the least value of a 2 a^2 is 3 3 and futher a 4 a^4 is 3 2 = 9 3^2 = 9 . Therefore the smallest value of a 4 + a 2 7 a^4 + a^2 - 7 is 9 + 3 7 = 5 9 + 3 - 7 = \boxed{5} .

0 pending reports

×

Problem Loading...

Note Loading...

Set Loading...